Limit problem bee

Calculus Level 3

lim x 0 1 + tan x 3 1 + sin x 3 x 3 = 1 k \large \displaystyle \lim_{x \to 0}\frac{\sqrt[3]{1+\tan{x}}-\sqrt[3]{1+\sin{x}}}{x^3} = \frac{1}{k}

Find the value of k k satisfies the equation above.


The answer is 6.

This section requires Javascript.
You are seeing this because something didn't load right. We suggest you, (a) try refreshing the page, (b) enabling javascript if it is disabled on your browser and, finally, (c) loading the non-javascript version of this page . We're sorry about the hassle.

5 solutions

Joseph Newton
Oct 10, 2018

Let 1 + tan x 3 = A \sqrt[3]{1+\tan x}=A and 1 + sin x 3 = B \sqrt[3]{1+\sin x}=B . L = lim x 0 1 + tan x 3 1 + sin x 3 x 3 = lim x 0 A B x 3 L=\lim_{x\to0}\frac{\sqrt[3]{1+\tan x}-\sqrt[3]{1+\sin x}}{x^3}=\lim_{x\to0}\frac{A-B}{x^3} Now, since A 3 B 3 = ( A B ) ( A 2 + A B + B 2 ) A^3-B^3=(A-B)(A^2+AB+B^2) : L = lim x 0 A B x 3 × A 2 + A B + B 2 A 2 + A B + B 2 = lim x 0 A 3 B 3 x 3 × 1 A 2 + A B + B 2 = lim x 0 ( 1 + tan x ) ( 1 + sin x ) x 3 × 1 A 2 + A B + B 2 \begin{aligned}L&=\lim_{x\to0}\frac{A-B}{x^3}\times\frac{A^2+AB+B^2}{A^2+AB+B^2}\\ &=\lim_{x\to0}\frac{A^3-B^3}{x^3}\times\frac1{A^2+AB+B^2}\\ &=\lim_{x\to0}\frac{(1+\tan x)-(1+\sin x)}{x^3}\times\frac1{A^2+AB+B^2}\end{aligned} When x = 0 , A = 1 x=0,\,A=1 and B = 1 B=1 . This means the right side is defined for x = 0 x=0 , and we can remove it from the limit: L = ( lim x 0 1 + tan x 1 sin x x 3 ) × 1 1 2 + 1 × 1 + 1 2 L = 1 3 lim x 0 tan x sin x x 3 \begin{aligned}L&=\left(\lim_{x\to0}\frac{1+\tan x-1-\sin x}{x^3}\right)\times\frac1{1^2+1\times1+1^2}\\ L&=\frac13\lim_{x\to0}\frac{\tan x-\sin x}{x^3}\end{aligned} Now we simply use L'Hospital's rule 3 times: L = 1 3 lim x 0 sec 2 x cos x 3 x 2 = 1 3 lim x 0 2 sec 2 x tan x + sin x 6 x = 1 3 lim x 0 2 sec 4 x + 4 sec 2 x tan 2 x + cos x 6 = 1 3 × 2 + 0 + 1 6 = 1 3 × 3 6 = 1 6 \begin{aligned}L&=\frac13\lim_{x\to0}\frac{\sec^2 x-\cos x}{3x^2}\\ &=\frac13\lim_{x\to0}\frac{2\sec^2 x\tan x+\sin x}{6x}\\ &=\frac13\lim_{x\to0}\frac{2\sec^4 x+4\sec^2 x\tan^2 x+\cos x}6\\ &=\frac13\times\frac{2+0+1}6\\ &=\frac13\times\frac36\\ &=\frac16\end{aligned} Therefore k = 6 \boxed{k=6}

Durian Ice
Oct 14, 2018

Using Taylor expansions as x 0 x\rightarrow 0 :

( 1 + x ) n = 1 + n x + O ( x ) \displaystyle{(1+x)^n=1+nx+O(x)}

tan x = x + 1 3 x 3 + O ( x 3 ) \displaystyle{\tan x=x+\frac{1}{3} x^3+O(x^3)}

sin x = x 1 6 x 3 + O ( x 3 ) \displaystyle{\sin x=x-\frac{1}{6} x^3+O(x^3)}

The limit becomes

lim x 0 1 + 1 3 tan x ( 1 + 1 3 sin x ) x 3 \displaystyle{\lim_{x\rightarrow 0} \frac{1+\frac{1}{3}\tan x-\left( 1+\frac{1}{3}\sin x \right)}{x^3}}

= 1 3 lim x 0 tan x sin x x 3 =\displaystyle{\frac{1}{3}\lim_{x\rightarrow 0} \frac{\tan x-\sin x}{x^3}}

= 1 3 lim x 0 x + 1 3 x 3 ( x 1 6 x 3 ) x 3 =\displaystyle{\frac{1}{3}\lim_{x\rightarrow 0} \frac{x+\frac{1}{3}x^3 -\left( x-\frac{1}{6}x^3 \right)}{x^3}}

= 1 6 =\displaystyle{\frac{1}{6}}

Akash Kumar Singh
Oct 12, 2018

We can use binomial expansion to solve the question

lim x 0 1 + 1 3 tan ( x ) 1 1 3 sin ( x ) x 3 \lim_{x\to 0}\frac{1+\frac{1}{3} \tan(x) -1-\frac{1}{3} \sin(x)}{x^3}

and apply L' Hospital's rule 3 times

Hello, I am very interested in your use of the binomial expansion. I tried to understand it online, but dont fully understand your way of doing it. Could you please develop your calculations? Thanks a lot!

Pierre Bongrand - 2 years, 8 months ago

Log in to reply

Hello, You can use binomial expression and omit the higher order terms the expression in ( 1 + t a n ( x ) ) 1 / 3 (1+tan(x)) ^{1/3} now you can apply binomial expansion to it. It will become ( 1 + 1 3 t a n ( x ) ) (1+\frac{1}{3} tan(x) ) by approximation here follows the explanation what I have done . https://math.stackexchange.com/questions/210110/whats-the-name-of-the-approximation-1xn-approx-1-xn hope it helped

Akash Kumar Singh - 2 years, 7 months ago

Thank you so much ! It is very clear now :)

Pierre Bongrand - 1 year, 11 months ago

Observe that from the algebraic identity: a 3 b 3 = ( a b ) ( a 2 + a b + b 2 ) a b = a 3 b 3 a 2 + a b + b 2 \begin{aligned}a^3 - b^3 &= (a-b)(a^2 + ab + b^2) \\ a-b &= \frac{a^3 - b^3}{a^2 + ab + b^2}\end{aligned} We could substitute a = 1 + tan x 3 a = \sqrt[3]{1 + \tan{x}} and b = 1 + sin x 3 b = \sqrt[3]{1 + \sin{x}} and by putting it into the limit, we get lim x 0 1 + tan x 3 1 + sin x 3 x 3 = lim x 0 ( 1 + tan x ) ( 1 + sin x ) x 3 1 1 + tan x 3 2 + ( 1 + sin x ) ( 1 + tan x ) 3 + 1 + sin x 3 2 = lim x 0 tan x sin x x 3 1 1 + tan x 3 2 + ( 1 + sin x ) ( 1 + tan x ) 3 + 1 + sin x 3 2 \begin{aligned}\lim_{x\to 0} \frac{\sqrt[3]{1 + \tan{x}} - \sqrt[3]{1 + \sin{x}}}{x^3} &= \lim_{x\to 0} \frac{(1 + \tan{x}) - (1 + \sin{x})}{x^3}\cdot \frac{1}{\sqrt[\frac{3}{2}]{1 + \tan{x}} + \sqrt[3]{(1 + \sin{x})(1 + \tan{x})} + \sqrt[\frac{3}{2}]{1 + \sin{x}}} \\ &= \lim_{x\to 0} \frac{\tan{x} - \sin{x}}{x^3}\cdot \frac{1}{\sqrt[\frac{3}{2}]{1 + \tan{x}} + \sqrt[3]{(1 + \sin{x})(1 + \tan{x})} + \sqrt[\frac{3}{2}]{1 + \sin{x}}}\end{aligned} The Maclaurin series for sin x \sin{x} and tan x \tan{x} is sin x = x x 3 3 ! + x 5 5 ! x 7 7 ! + tan x = x + x 3 3 + 2 x 5 15 + 17 x 7 315 + \begin{aligned} \sin{x} &= x - \frac{x^3}{3!} + \frac{x^5}{5!} - \frac{x^7}{7!} + \dots \\ \tan{x} &= x + \frac{x^3}{3} + \frac{2x^5}{15} + \frac{17x^7}{315} + \dots \end{aligned} and using them to the left side fraction in the limit gives us = lim x 0 ( x + x 3 3 + 2 x 5 15 + 17 x 7 315 + ) ( x x 3 3 ! + x 5 5 ! x 7 7 ! + ) x 3 1 1 + tan x 3 2 + ( 1 + sin x ) ( 1 + tan x ) 3 + 1 + sin x 3 2 = lim x 0 1 2 x 3 + ( 2 15 1 5 ! ) x 5 + x 3 1 1 + tan x 3 2 + ( 1 + sin x ) ( 1 + tan x ) 3 + 1 + sin x 3 2 = lim x 0 ( 1 2 + ( 2 15 1 5 ! ) x 2 + ) 1 1 + tan x 3 2 + ( 1 + sin x ) ( 1 + tan x ) 3 + 1 + sin x 3 2 \begin{aligned} &= \lim_{x\to 0} \frac{\left(x + \frac{x^3}{3} + \frac{2x^5}{15} + \frac{17x^7}{315} + \dots\right) - \left(x - \frac{x^3}{3!} + \frac{x^5}{5!} - \frac{x^7}{7!} + \dots \right)}{x^3}\cdot \frac{1}{\sqrt[\frac{3}{2}]{1 + \tan{x}} + \sqrt[3]{(1 + \sin{x})(1 + \tan{x})} + \sqrt[\frac{3}{2}]{1 + \sin{x}}} \\ &= \lim_{x\to 0} \frac{\frac{1}{2}x^3 + \left(\frac{2}{15} - \frac{1}{5!}\right)x^5 + \dots}{x^3}\cdot \frac{1}{\sqrt[\frac{3}{2}]{1 + \tan{x}} + \sqrt[3]{(1 + \sin{x})(1 + \tan{x})} + \sqrt[\frac{3}{2}]{1 + \sin{x}}} \\ &= \lim_{x\to 0} \left(\frac{1}{2} + \left(\frac{2}{15} - \frac{1}{5!}\right)x^2 + \dots \right)\cdot \frac{1}{\sqrt[\frac{3}{2}]{1 + \tan{x}} + \sqrt[3]{(1 + \sin{x})(1 + \tan{x})} + \sqrt[\frac{3}{2}]{1 + \sin{x}}} \end{aligned} We could substitute x = 0 x=0 to the expression of the limit, it evaluates to 1 6 \frac{1}{6} and hence k = 6 \boxed{k=6} .

Hybrid M
Oct 11, 2018

tan x = 1 + x 3 3 \frac{x^3}{3} + 2 x 5 15 \frac{2x^5}{15} + ...

sin x = x - x 3 6 \frac{x^3}{6} + x 5 120 \frac{x^5}{120} - ...

(1 + tan x)^(1/3) - (1 + sin x)^(1/3) = x 3 6 \frac{x^3}{6} + higher powers of x (using binomial expansion as |tan x| < 1 and |sin x| < 1 as x tends to 0)

Hence, the required limit is 1/6

0 pending reports

×

Problem Loading...

Note Loading...

Set Loading...